Simplifying Complicated Trigonometric Integrals

Click For Summary
The discussion revolves around simplifying the integral ∫|y|ds for the curve defined by (x²+y²)²=2²(x²-y²). The user attempted to convert the integral into polar coordinates, leading to a complex expression involving trigonometric functions. They expressed concern over the difficulty of the resulting integral after substitution, indicating a potential error in their calculations. Another participant suggested tackling a simpler related integral to build confidence and understanding. The conversation emphasizes the challenges of trigonometric integrals and the importance of methodical problem-solving.
ace1412
Messages
1
Reaction score
0

Homework Statement



\int_{C}|y|ds where C is the curve (x^{2}+y^{2})^{2}=2^{2}(x^{2}-y^{2})

Homework Equations


The Attempt at a Solution



i used polar coordinates x = r cos \theta and y = r sin \theta

then substituted into the equation to get r = 2\sqrt{cos 2\theta}

since r\geq0 gives -\frac{\pi}{4}\leq \theta\leq \frac{\pi}{4}

substituting back gives x = 2\sqrt{cos 2\theta}cos \theta and y = 2\sqrt{cos 2\theta}sin \theta

then i calculated \frac{dx}{d\theta}=-\frac{2sin 2\theta cos \theta}{\sqrt{cos 2\theta}}-\frac{2sin \theta}{\sqrt{cos 2\theta}} and \frac{dy}{d\theta}=-\frac{2sin 2\theta sin \theta}{\sqrt{cos 2\theta}}+\frac{2cos \theta}{\sqrt{cos 2\theta}}

and found ds=\sqrt{(\frac{dx}{d\theta})^{2}+(\frac{dy}{d\theta})^{2}}d\theta=(4tan 2\theta sec 2\theta+4 cos 2\theta)d\theta

now substituting back to the integral gives 8\int^{-\frac{\pi}{4}}_{\frac{\pi}{4}}\sqrt{cos 2\theta}|sin \theta|(tan 2\theta sec 2\theta+ cos 2\theta)d\theta

which looks terribly difficult, so i inferred that i did something wrong somewhere, can someone please shed a bit of light? thanks
 
Last edited:
Physics news on Phys.org
Why not just muscle through it? I mean suppose you had to? Could you? Start with this piece:

\int \frac{\sin(t) \sin(2t)}{(\cos(2t)^{3/2}}dt

How about just that part? Already has the 2t thing in the top and bottom. Maybe start with parts. Keep working through it. See what happens.
 
Question: A clock's minute hand has length 4 and its hour hand has length 3. What is the distance between the tips at the moment when it is increasing most rapidly?(Putnam Exam Question) Answer: Making assumption that both the hands moves at constant angular velocities, the answer is ## \sqrt{7} .## But don't you think this assumption is somewhat doubtful and wrong?

Similar threads

  • · Replies 4 ·
Replies
4
Views
1K
  • · Replies 14 ·
Replies
14
Views
2K
  • · Replies 5 ·
Replies
5
Views
2K
  • · Replies 19 ·
Replies
19
Views
2K
Replies
3
Views
2K
  • · Replies 20 ·
Replies
20
Views
2K
  • · Replies 3 ·
Replies
3
Views
2K
  • · Replies 8 ·
Replies
8
Views
2K
  • · Replies 14 ·
Replies
14
Views
2K
  • · Replies 3 ·
Replies
3
Views
2K